Board logo

標題: 101北市中正高中 [打印本頁]

作者: 八神庵    時間: 2012-6-17 16:59     標題: 101北市中正高中

如題
請享用
---------
題外話
小弟今年沒考
因為暑假要去高雄進修
等我進修回來還要補去年六月底以後到今年的進度
可能會仆街orz
各位多加油

附件: 北市中正高中.pdf (2012-6-17 16:59, 206.02 KB) / 該附件被下載次數 13960
https://math.pro/db/attachment.php?aid=1259&k=804dba6b15eda1c6f586c0f6c75f0d6a&t=1714092907
作者: arend    時間: 2012-6-17 18:14

請教第三題
還有第四題m=-3/2怎麼得到的

版上高手請不吝告知

另外第二題
先令高h, C到山腳為x
我是利用tan(3theta)=tan(2theta+theta)
              tan(2theta)=tan(theta+theta)
              得x=35 在求出h

可是我這樣做就花了我約20分鐘
請教有更簡潔的作法嗎

謝謝
作者: 老王    時間: 2012-6-17 18:45     標題: 回復 2# arend 的帖子

第二題
如圖,可知 \( CD=AD=100 \) ,
在 \( \Delta CDE \) 中用正弦定理, 得到
\(\displaystyle \frac{100}{\sin3\theta}=\frac{40}{\sin\theta} \)
\(\displaystyle 3-4\sin^2\theta=\frac{5}{2} \)
\(\displaystyle \sin^2\theta=\frac{1}{8} \)
\(\displaystyle \sin\theta=\frac{1}{\sqrt8} \)
\(\displaystyle \cos\theta=\frac{\sqrt7}{\sqrt8} \)
\(\displaystyle BC=CD\sin2\theta=25\sqrt{7} \)


第三題
若第一次反面,則只要剩下的 \( n-1 \) 次不要連續正面就好;
若第一次正面,則第二次必須反面,只要剩下的 \( n-2 \) 次不要連續正面就好;
故 \(\displaystyle P_n=\frac{1}{2}P_{n-1}+\frac{1}{4}P_{n-2} \)
以及 \( P_1=1, P_2=\frac{3}{4} \)
剩下的就慢慢算。

第四題
前者兩根為 \( 1+i, 1-i \)
如果後者有共軛虛根,我們知道虛根對稱於 \( x \) 軸,只要實部不為 \( 1 \) ,必為等腰梯形,四點共圓;
\(\displaystyle D=m^2-1<0, \Rightarrow -1 < m < 1 \) ;
實部為 \( 1 \) ,\( -2m=2 \) 即 \( m=-1 \) 不在判別式的範圍內,所以這部分是 \( -1 < m < 1 \) 。
如果是相異實根,設兩根為 \( p < q \) ,顯然要有 \( p < 1 < q \) 才行,
而且直徑會在 \( x \) 軸上,由直角三角形子母相似性質知道 \( (1-p)(q-1)=1^2 \)
\( -(1+2m+1)=1 \)
\(\displaystyle m=-\frac{3}{2} \)

[ 本帖最後由 老王 於 2012-6-17 07:48 PM 編輯 ]

圖片附件: 101中正高中填充2.jpg (2012-6-17 18:57, 10.67 KB) / 該附件被下載次數 9213
https://math.pro/db/attachment.php?aid=1260&k=10cffbbf0f1f8f0f60eabb1d8e476f61&t=1714092907


作者: arend    時間: 2012-6-17 18:53

引用:
原帖由 老王 於 2012-6-17 06:45 PM 發表
如圖,可知 \( CD=AD=100 \) ,
在 \( \Delta CDE \) 中用正弦定理, 得到
\(\displaystyle \frac{100}{\sin3\theta}=\frac{40}{\sin\theta} \)
\(\displaystyle 3-4\sin^2\theta=\frac{5}{2} \) ...
謝謝老王老師

真是太漂亮的解法

再次感謝你
作者: bugmens    時間: 2012-6-17 19:14

1.
設\( Γ_1 \):\( \displaystyle \frac{x^2}{a^2}+\frac{y^2}{b^2} \le 1 \)、\( Γ_2 \):\( \displaystyle \frac{(x-a)^2}{a^2}+\frac{y^2}{b^2} \le 1 \),其中\( a>b>0 \),求\( Γ_1 \)與\( Γ_2 \)交集的區域面積為?
看到老王老師的解法,讓我想到另一題

通過橢圓\( \displaystyle \frac{x^2}{25}+\frac{y^2}{16}=1 \)上兩點\( (0,-4) \),\( \displaystyle (\frac{5 \sqrt{3}}{2},2) \)的直線L,將橢圓內部分割成兩個區域,試問較小區域的面積為?
(1)\( \displaystyle \frac{20 \pi}{3} \) (2)\( \displaystyle \frac{25 \pi}{3}-\frac{25 \sqrt{3}}{4} \) (3)\( \displaystyle \frac{20 \pi}{3}-\frac{25 \sqrt{3}}{4} \) (4)\( \displaystyle \frac{20 \pi}{3}-5 \sqrt{3} \)
(98桃園縣國中聯招,https://math.pro/db/thread-826-1-1.html)


8.若\(\cases{\displaystyle x=\frac{12z^2}{1+36z^2} \cr y=\frac{12x^2}{1+36x^2} \cr z=\frac{12y^2}{1+36y^2}} \),則\( x+y+z= \)

解方程組\( \displaystyle \cases{1+x^2=2y \cr 1+y^2=2z \cr 1+z^2=2x} \)。


102.3.28補充
Find all real solutions to the following system of equations. Carefully justify your answer.
\( \cases{\displaystyle \frac{4x^2}{1+4x^2}=y \cr \frac{4y^2}{1+4y^2}=z \cr \frac{4z^2}{1+4z^2}=x} \)
(1996 Canada National Olympiad,http://www.artofproblemsolving.c ... id=51&year=1996)
https://math.stackexchange.com/q ... -involving-fixed-po

計算題
4.
已知函數\( f(x)=ax^2-c \)( \( a,c \in R \) )滿足\( -4 \le f(1) \le -1 \),\( -1 \le f(2) \le 5 \),
(1)利用Lagrange多項式,將\( f(x) \)表為\( P_1(x)f(1)+P_2(x)f(2) \),其中\( P_1(x) \)與\( P_2(x) \)均為二次多項式,則\( P_1(x)= \)?\( P_2(x)= \)?
(2)求\( f(3) \)之值的範圍?

高中數學常見題之一題多解
http://i.imgur.com/XaQ6H.gif
http://i.imgur.com/MvBFJ.gif
http://i.imgur.com/HDRBC.gif

已知a、b為實數,\( f(x)=ax^2+bx \),滿足\( 1 \le f(1) \le 2 \),\( 2 \le f(2) \le 4 \),若\( P \le f(3) \le Q \),則數對\( (P,Q) \)為何?
(101桃園縣高中聯招,https://math.pro/db/thread-1416-1-1.html)

101.11.25補充
設\( f(x)=ax^2+bx+c \),若已知\( 1 \le f(1) \le 2 \),\( 1 \le f(2) \le 4 \),\( 3 \le f(3) \le 11 \),求\( f(4) \)之最大值?
h ttp://forum.nta.org.tw/examservice/showthread.php?t=12657(連結已失效)

110.8.2補充
若二次實係數多項式函數\(f(x)\)滿足\(\cases{-1\le f(1)\le 3 \cr 6 \le f(2)\le 10 \cr 2 \le f(4) \le 24}\),則\(f(7)\)的最大值?
(110竹東高中,https://math.pro/db/thread-3533-1-1.html)

6.
設\( x_1 \),\( x_2 \),…,\( x_n \)都是正數且\( n \ge 2 \),試分別利用算幾不等式與數學歸納法兩種方法證明:
\( \displaystyle \frac{x_1^2}{x_2}+\frac{x_2^2}{x_3}+\frac{x_3^2}{x_4}+……+\frac{x_{n-1}^2}{x_n}+\frac{x_n^2}{x_1}\ge x_1+x_2+…+x_n \)

設\( x_1,x_2,...,x_n \)都是正數,試證\( \displaystyle \frac{x_1^2}{x_2}+\frac{x_2^2}{x_3}+...+\frac{x_{n-1}^2}{x_n}+\frac{x_n^2}{x_1}\ge x_1+x_2+...+x_n \)。
(100桃園高中,https://math.pro/db/thread-1144-1-1.html)

設\( a_1,a_2,...,a_n \)皆為正數,求證:\( \displaystyle \sum_{k=1}^n a_k \le \frac{a_1^2}{a_2}+\frac{a_2^2}{a_3}+...+\frac{a_n^2}{a_1} \)
(94高中數學能力競賽 台南區筆試一試題,h ttp://www.math.nuk.edu.tw/senpe ... _High_Tainan_01.pdf連結已失效)

101.12.11補充
我在這本書找到這題的數學歸納法證明
夏興國,數學歸納法縱橫談

圖片附件: 數學奧林匹克教程.gif (2012-6-17 19:14, 10.68 KB) / 該附件被下載次數 10436
https://math.pro/db/attachment.php?aid=1261&k=e36639e4e80dc0190f00f4ffb121239d&t=1714092907



附件: 101中正高中-計算6數學歸納法解題.zip (2012-12-11 15:27, 15.18 KB) / 該附件被下載次數 11507
https://math.pro/db/attachment.php?aid=1477&k=c7e2b971ce1e22a6835da7b654d669b0&t=1714092907
作者: bluewing    時間: 2012-6-17 21:20

老師您好,請問填充第1題兩橢圓所夾面積應該要怎麼處理呢?
可以指點一下嗎?謝謝您。
作者: 老王    時間: 2012-6-17 21:45     標題: 回復 6# bluewing 的帖子

利用伸縮變換,作出兩圓 \( x^2+y^2=a^2 \) 和 \( (x-a)^2+y^2=a^2 \)
計算兩圓交集面積再乘上 \(\displaystyle  \frac{b}{a} \) 就好。
如圖
兩圓交集面積為 \(\displaystyle 2 \times \frac{a^2\pi}{3}-\frac{\sqrt{3}a^2}{2} \)
所以所求為 \(\displaystyle \frac{2ab\pi}{3}-\frac{\sqrt{3}ab}{2} \)

[ 本帖最後由 老王 於 2012-6-17 09:46 PM 編輯 ]

圖片附件: 101中正高中填充1.jpg (2012-6-17 21:46, 18.71 KB) / 該附件被下載次數 9077
https://math.pro/db/attachment.php?aid=1264&k=78a7a3bcfb05128bd07d6edf03a1cb43&t=1714092907


作者: bluewing    時間: 2012-6-18 19:24

老師您好,謝謝您的解答,很詳細...
另外可以請問計算4的第1小題應該如何處理呢??
只有兩個點的值怎麼用lagrange插值法呢?謝謝您。
作者: tsusy    時間: 2012-6-18 19:53     標題: 回復 8# bluewing 的帖子

計算 4. 沒看過這樣的變形的 Lagrange 插值多項式

不過猜測 \(\displaystyle P_{1}(x)=\frac{1}{(-3)}(x^{2}-4) \), \(\displaystyle P_{2}(x)=\frac{1}{3}(x^{2}-1) \)

也就是滿足次數 2, 一次項零,1, 2 代入又會等於 1,0 的多項式
作者: 老王    時間: 2012-6-18 21:38     標題: 回復 8# bluewing 的帖子

我的想法是,如果找到某個 \( k \) ,使得 \( f(k)=0 \)
那麼
\(\displaystyle f(x)=f(1) \frac{(x-2)(x-k)}{(1-2)(1-k)}+f(2) \frac{(x-1)(x-k)}{(2-1)(2-k)} \)
但是對於找 \( f(3) \) 的範圍似乎沒有助益。
作者: childgrow    時間: 2012-6-18 23:37     標題: 想請教計算2、計算3(3)、計算6

謝謝各位老師了
作者: tsusy    時間: 2012-6-19 00:09     標題: 回復 11# childgrow 的帖子

計算 2. 不要被嚇到了,題目都叫我們猜了,當然是列個幾項猜答案,然後證明之。

\( n=2\), \( 1225=35^2 \)

\( n=3 \), \( 112225=335^2 \)

看到這已經猜出答案了。

以完全平方公式計算得 \( 333..335^2  = 333..33 \times 333..34\times 10^2 +25 \)

前面相乘補一個 \( \frac33 \) 給它得 \( 999..99\times 333..34\div 3\)

\( 999..99 \) 寫成 \( 1000..00-1 \) 分配乘開得 \( (333..34000..00-333..34)\div 3 = 333..33666..66\div 3 =111..11222..22 \)

補上兩個 0 加 25,就得 \( 333..335^2  = 111..11222..2225 \)

註:以上所有的 aaa..aa 長度一樣長
作者: sanghuan    時間: 2012-6-19 17:35

想請問各位老師填充題第六題
方程式\( ax^2-4ax +1= 0\) 的兩個正數解\( \alpha,\beta\)滿足不等式\(|log\alpha -log\beta | \le 1\),則實數a 的範圍為__________

不知道a的上界是怎麼求得的   謝謝大家

[ 本帖最後由 sanghuan 於 2012-6-20 01:03 PM 編輯 ]
作者: 阿光    時間: 2012-6-20 13:50

想請教計算第1(2)(3),第3(3) ,第4(1) ,謝謝
作者: sanghuan    時間: 2012-6-20 14:18

引用:
原帖由 阿光 於 2012-6-20 01:50 PM 發表
想請教計算第1(2)(3),第3(3) ,第4(1) ,謝謝
計算1(2)(3)用內積定義去想

計算3(3)我是用過P坐一平行於 \( \overline{BC} \)的線 去看比例關係 (先求出 \( \overline{PL} \)、\( \overline{PM} \)、\( \overline{PN} \))

先試試看吧   真的不行我再詳細PO

[ 本帖最後由 sanghuan 於 2012-6-20 02:21 PM 編輯 ]
作者: yaung    時間: 2012-6-24 18:11     標題: 回復 15# sanghuan 的帖子

計算3(3)比例關係知道也求出 PL、PM、PN,但還是算不出來~可否請你詳細PO呢?謝謝
作者: tsusy    時間: 2012-6-24 20:30     標題: 回復 16# yaung 的帖子

一般的任意比例的確,的確很難算。

但這題的是特殊比例,應該關注 \( 2 \overline{PL}: 3\overline{PM}: 4\overline{PN}=1:1:1 \)

再回到 (1) 時,就知道這三個量的意義了
作者: sanghuan    時間: 2012-6-24 21:25     標題: 回復 16# yaung 的帖子

承寸絲老師所說可知

其實三角形APB、APC、BPC的面積相等且等於 三分之一的三角形ABC之面積

可知若過P點坐一平行於\( \overline{BC} \)之線交\( \overline{AB} \)於O、交\( \overline{AC} \)於O'

則\( \overline{AO} \):\( \overline{AB} \) = \( \overline{AO'} \):\( \overline{AC} \) = 1:3 ,因此\( \overline{AO} \)和\( \overline{AO'} \)可知

又在三角形AOP中  兩倍三角形AOP之面積 = \( \overline{AO} \)X\( \overline{PL} \) = \( \overline{OP} \) X (A到\( \overset { \rightharpoonup  }{ OP }  \) 的距離)

其中 A到\( \overset { \rightharpoonup  }{ OP }  \) 的距離可由三角形ABC與PBC之面積關係求得

因此\( \overline{OP} \)可得   同理\( \overline{PO'} \)亦可得  

又O、P、O'在同一直線上 可得\( \overline{AP} \) 和\( \overline{AO} \)、\( \overline{AO'} \)的關係

再把\( \overline{AO} \)、\( \overline{AO'} \)分別轉為\( \overline{AB} \)、\( \overline{AC} \)即可




我是覺得這個方法很麻煩   不過可解就是了  給大家參考    不知道有沒有更快了方法

[ 本帖最後由 sanghuan 於 2012-6-24 09:27 PM 編輯 ]
作者: jmfeng2001    時間: 2012-6-24 21:42

承上...
當三角形APC,APB,BPC面積相等...
不是重心嗎...
還是我解讀有誤
作者: sanghuan    時間: 2012-6-24 21:47

引用:
原帖由 jmfeng2001 於 2012-6-24 09:42 PM 發表
承上...
當三角形APC,APB,BPC面積相等...
不是重心嗎...
還是我解讀有誤
哈哈   沒想到   其實上面那個方法是我還沒聯想到面積相等時做的解答  受教啦

[ 本帖最後由 sanghuan 於 2012-6-24 09:54 PM 編輯 ]
作者: jmfeng2001    時間: 2012-6-24 22:00

甭客氣了...
我也是考完...回來想很久才想到...
在這裡...一直向各位老師學習...
很感謝大家...
一起研究吧
作者: sanghuan    時間: 2012-6-24 22:11

引用:
原帖由 jmfeng2001 於 2012-6-24 10:00 PM 發表
甭客氣了...
我也是考完...回來想很久才想到...
在這裡...一直向各位老師學習...
很感謝大家...
一起研究吧
大家互相幫忙   一起加油!!!!
作者: shingjay176    時間: 2012-7-10 11:06     標題: 回復 1# 八神庵 的帖子

計算題第一題。第一小題。第二小題
再寫過一次‧原本的答案有誤

[ 本帖最後由 shingjay176 於 2013-1-11 10:23 PM 編輯 ]
作者: andyhsiao    時間: 2012-7-10 11:49

引用:
原帖由 sanghuan 於 2012-6-19 05:35 PM 發表
想請問各位老師填充題第六題
方程式\( ax^2-4ax +1= 0\) 的兩個正數解\( \alpha,\beta\)滿足不等式\(|log\alpha -log\beta | \le 1\),則實數a 的範圍為__________

不知道a的上界是怎麼求得的   謝謝大家 ...
參考看看^^..

圖片附件: 3.jpg (2012-7-10 11:49, 52.03 KB) / 該附件被下載次數 5443
https://math.pro/db/attachment.php?aid=1374&k=1989536074bda8188fcdc92b93b0e393&t=1714092907


作者: redik    時間: 2012-7-22 20:43

引用:
原帖由 shingjay176 於 2012-7-10 11:06 AM 發表
計算題第一題。第一小題。第二小題
第三小題。符合題意的時候,三角形PAQ為等腰三角形,AO是底邊PQ上的中垂線。因此面積為
{4*sqrt(14)/2}=2√14
是這樣的,關於計算題有幾個問題想問:

1.計算1第2小題關於求最大值的部分

我的作法如下:




2.計算6的證明方法,說要用算幾不等式,目前實在沒有頭緒...orz

3.計算5我只有解出第一題,第二題解t時,解出這個方程式:64t^4+64t^3+37t^2-27t-27=0,好像解不出來?

懇請各位老師賜教了 orz
作者: Ellipse    時間: 2012-7-22 23:08

引用:
原帖由 redik 於 2012-7-22 08:43 PM 發表

是這樣的,關於計算題有幾個問題想問:
2.計算6的證明方法,說要用算幾不等式,目前實在沒有頭緒...orz
計算6
[(x1)^2+(x2)^2]/ x2  +[(x2)^2+(x3)^2]/ x3  +......................+[(xn)^2+(x1)^2]/ x1

>= (2*x1*x2)/x2 + (2*x2*x3)/x3+......................+(2*xn*x1)/x1        

[註: [(x1)^2+(x2)^2]/2 >= [(x1)^2*(x2)^2]^0.5  
=>  (x1)^2+(x2)^2 >= 2*x1*x2  就用到算幾不等式 ]

[(x1)^2/x2  + x2 ] + [(x2)^2/x3  + x3 ] +..................+[(xn)^2/x1  + x1 ]

>= 2*x1+2*x2+...................+2*xn   

可得
(x1)^2/x2 + (x2)^2/x3  +..................+(xn)^2/x1

>= x1+x2+...................+ xn

[ 本帖最後由 Ellipse 於 2012-7-22 11:09 PM 編輯 ]
作者: Ellipse    時間: 2012-7-22 23:46

引用:
原帖由 redik 於 2012-7-22 08:43 PM 發表

是這樣的,關於計算題有幾個問題想問:
3.計算5我只有解出第一題,第 ...
計算3
算到後面
t^4+t^3+t^2 =t^2*(t^2+t+1)=27/(4a^2) ----------------(1)
t^2+t+1=16/a^2 ------------------(2)
將(2)代入(1)
t^2*(16/a^2)=27/(4a^2)
因為a,b,c>0 且t=c/a>0
因此 t^2 =27/64
t=3*3^0.5/8
(您的四次方程式有一個解就是這個答案,其它不合)
再代入(2)解a
不過a的數據很醜

有算錯請指正
作者: Ellipse    時間: 2012-7-22 23:50

引用:
原帖由 redik 於 2012-7-22 08:43 PM 發表

是這樣的,關於計算題有幾個問題想問:

1.計算1第2小題關於求最大值的部分

我的作法如下:

個人認為您的算法應該是ok的

shingjay176老師這題最一小題寫的方式可能需要再檢驗或再仔細說明一下
作者: andyhsiao    時間: 2012-7-24 15:22

引用:
原帖由 redik 於 2012-7-22 08:43 PM 發表

是這樣的,關於計算題有幾個問題想問:

1.計算1第2小題關於求最大值的部分

我的作法如下:
你的是對的...
shingjay176的5/9答案是計算錯誤來的....(他把4根號14的平方算錯...才會有這個答案)
不過這不是最大問題...他的算法並不能保證P和Q一定在XY平面上...
他只能保證PQ是通過原點為圓心半徑為2的圓上的點...

我用另外一個方法算..參考看看

圖片附件: 1.png (2012-7-24 15:22, 123.25 KB) / 該附件被下載次數 5110
https://math.pro/db/attachment.php?aid=1409&k=ca0fe06d8a4c761f3ee843faf6d05c16&t=1714092907


作者: andyhsiao    時間: 2012-7-24 17:47

計算2...參考看看

圖片附件: 2.png (2012-7-24 17:47, 76.08 KB) / 該附件被下載次數 5223
https://math.pro/db/attachment.php?aid=1410&k=dd362fe2c685a8bb4da04f40d8b32e7e&t=1714092907


作者: redik    時間: 2012-7-24 22:46

引用:
原帖由 Ellipse 於 2012-7-22 11:08 PM 發表


計算6
[(x1)^2+(x2)^2]/ x2  +[(x2)^2+(x3)^2]/ x3  +......................+[(xn)^2+(x1)^2]/ x1

>= (2*x1*x2)/x2 + (2*x2*x3)/x3+......................+(2*xn*x1)/x1        

[註: [(x1)^2+(x2)^2]/2 >= [ ...
1.厲害!我實在想不到這麼簡單就出來了....orz

2.第5題也是臨門一腳,感謝兩位老師

3.另外第6題漏看一個還需要數學歸納法的證明,n=1的部分沒問題

但是 n=k 到 n=k+1 的部分就有困難了,怎麼兜都兜不出來 orz,不知道各位老師有沒有想法....

4.其他題目因為沒看到答案,懇請各位幫忙看一下有沒有錯@@

計算3 : (1) 3*sqrt(15)/2    (2) 45/4  (3) (1/3,1/3)

計算4 : (1)P1(x) = (-x^2+4)/3,P2(x) = (x^2-1)/3   (2) -1<= f(3) <= 20

謝謝!
作者: natureling    時間: 2012-11-7 20:33     標題: 回復 17# tsusy 的帖子

@@...可以請問一下如何看出2PL:3PM:4PN=1:1:1...感恩
作者: natureling    時間: 2012-11-7 21:22     標題: 回復 9# tsusy 的帖子

@@可以請教一下tsusy大嗎??為何如何猜測@@...
作者: tsusy    時間: 2012-11-7 22:01     標題: 回復 32# natureling 的帖子

計算3

因為前面一段 柯西不等式沒人寫。

由面積和柯西不等式可得在 \( 2\overline{PL}=3\overline{PM}=4\overline{PN} \) 時會發生極值

後面由面積得到重心的,前面已有人討論過了。

回復 33# natureling 的帖子

計算3

一般我們將 Lagrange 插值多項式寫作 \( \sum c_k p_k(x) \) 之型式,其中 \( p_k \) ...(不會形容)

但如果不把 \( p_k \) 的式子詳細寫下了,我們還是知道 \( p_k(x_l) = \delta_{kl} \),其中 \( \delta_{kl}\)  為 Kronecker  記號。

所以其實的我猜測,是將 \( p_k \) 的性質推廣過來,而反 \( p_k \) 的形狀
作者: Sandy    時間: 2012-12-2 02:34     標題: 回復 32# natureling 的帖子

計算3
想請問計算5的想法,謝謝

圖片附件: 101中正高中.png (2012-12-2 02:34, 50.42 KB) / 該附件被下載次數 5049
https://math.pro/db/attachment.php?aid=1474&k=313caf26374504d940d8a1e9aa80088a&t=1714092907


作者: dream10    時間: 2012-12-2 07:36

計算5請參考
http://www.shiner.idv.tw/teachers/viewtopic.php?f=53&t=2936
作者: shingjay176    時間: 2013-1-11 12:42

這一題只有算出1/2的答案,0那組答案怎麼來的???

[ 本帖最後由 shingjay176 於 2013-1-11 12:48 PM 編輯 ]

圖片附件: [填充題第八題] 20130111_124709-1.jpg (2013-1-11 12:48, 141.4 KB) / 該附件被下載次數 5570
https://math.pro/db/attachment.php?aid=1497&k=e29d8673eaf01e392fc0ef7067f0145d&t=1714092907


作者: weiye    時間: 2013-1-11 13:18     標題: 回復 37# shingjay176 的帖子

你所寫的解答的第一步「將 \(x\) 倒數變成 \(\displaystyle \frac{1}{x}\) 」的先決要件是 \(x\) 不是 \(0\),




若 \(x=0\),由題述的第二式可得 \(y=0\),再由題述的第三式可得 \(z=0\),

同理,可知 \(x,y,z\) 只要三者有一數為零,則三數皆為零,因此 \(x+y+z=0\)。

若三數皆非零,就可以如你的解答步驟接續下去。
作者: kittyyaya    時間: 2013-1-31 01:34

想請問老王老師在#3發表的填充3解答
我的解法如下
x^2-(1/2)x-(1/4)=0
.....
x=(1+ - 根號5)/4
P_n=[(1+ 根號5)/4]^n * a+[(1- 根號5)/4]^n *b
這樣算下去 過程很可怕 請問老師們 我要如何做 謝謝
作者: weiye    時間: 2013-1-31 10:50     標題: 回復 39# kittyyaya 的帖子

填充第 3 題:

連續丟擲 \(n\) 回硬幣,在所有情況中,正面不連續出現的情況有 \(a_n\) 種,

則 \(a_1=2,a_2=3\),\(a_n=a_{n-1}+a_{n-2}\)


\(n\)\(1\)\(2\)\(3\)\(4\)\(5\)\(6\)\(7\)\(8\)\(9\)\(10\)
\(a_n\)\(2\)\(3\)\(5\)\(8\)\(13\)\(21\)\(34\)\(55\)\(89\)\(144\)


(是的~它是 Fibonacci 數列~:P)

所求=\(\displaystyle \frac{144}{2^{10}}=\frac{9}{64}\)


另解,

\(10\) 次中沒有正面的有 \(1\) 種,

\(10\) 次中恰有 \(1\) 次正面的有 \(C^{10}_1\) 種,

\(10\) 次中恰有 \(2\) 次正面(任兩正面都不相鄰)的有 \(C^9_2\) 種,

\(10\) 次中恰有 \(3\) 次正面(任兩正面都不相鄰)的有 \(C^8_3\) 種,

\(10\) 次中恰有 \(4\) 次正面(任兩正面都不相鄰)的有 \(C^7_4\) 種,

\(10\) 次中恰有 \(5\) 次正面(任兩正面都不相鄰)的有 \(C^6_5\) 種,

所求=\(\displaystyle \frac{1+C^{10}_1+C^9_2+C^8_3+C^7_4+C^6_5}{2^{10}}=\frac{9}{64}\)
作者: rock    時間: 2013-3-25 10:44

bugmen老師您的這段說明:計算第6

我在這本書找到這題的數學歸納法證明
夏興國,數學歸納法縱橫談



夏興國老師的文章中內容中 他有說到"優化假設"

我的看法是  當數學歸納法最後需要比較出x_1   x_n   x_n+1這個計算後的大小值時

如果假設這三數的大小順序(例如假設x_1 >  x_n >  x_n+1)   有可能造成不等式的不成立

所以我不懂為何他可以假設x_n+1為最大值

(還是我誤解他的說明??)


謝謝回答
作者: tsusy    時間: 2013-3-25 14:16     標題: 回復 41# rock 的帖子

這就像是平常寫證明的時候,從對稱性可以加入一些不失一般性的假設

\(x_1, x_2,x_3,\ldots, x_{n+1} \) 如果最大的是 \( x_5 \) 重新排序

\( x_6, x_7, x_8,\ldots, x_{n+1}, x_1, x_2, \ldots, x_5 \) 重新命名為 \( y_1, y_2, \ldots, y_{n+1} \)

那麼 \( y_{n+1} \) 就是最大的啦

而對 \({x_i}\), \( {y_i} \) 來說,不等式的兩邊值是相同的

故僅須對 \( {y_i} \) 去證明原命題
作者: rock    時間: 2013-3-26 10:49

我欠缺的就是這個解釋    非常清楚    太感謝了!
作者: 阿吉    時間: 2013-5-25 05:47     標題: 回復 31# redik 的帖子

計算4
只看到答案, 沒看到過程, 所以我來補過程:
f(x)是偶函數, 所以y=f(x)的圖形通過(1,f(1)), (-1,f(1)), (2,f(2))三點
由lagrange插值公式即可得到一多項式
最後整理後即可得到 P1(x)和P2(x)

ps 由f(1)=a-c和f(2)=4a-c很容易得到答案
但因為題目要求用lagrange插值公式
所以我猜應該要這麼寫(其實我原本的式子更醜, 我原本第三個點是用( sqrt(c/a) , 0 )   QQ

)


作者: cefepime    時間: 2016-9-29 23:38

計算題

1. A (3,1,2) 為空間中一點,P、Q 為 xy 平面上以原點 O 為圓心且半徑為 2 的圓之直徑兩端點,試計算下列問題:

(1) 內積 AP.AQ =?

(2) cos∠PAQ 的最大值與最小值。

(3) cos∠PAQ 為最小值時,ΔPAQ 的面積為何?


解:

(1) 所求 = AO² - r² = 14 - 4 = 10

(2)(3) 雖然題意暗示用內積,但亦可利用幾何性質來解。

平面上給定一線段 PQ (中點為 O),滿足∠PAQ 為定值 [範圍: (0, 180°)] 的動點 A,其構成對稱於 PQ 的兩圓弧。再思考當∠PAQ 變化時,兩圓弧的動態變化。依此考慮 AO 為定值時,∠PAQ 之值,可得如下結論:

1. 若 AO > PO ("A 在圓外"),則 AO 與 PQ 夾角 (取直角或銳角) 愈大,∠PAQ 愈大

2. 若 AO < PO ("A 在圓內"),則 AO 與 PQ 夾角 (取直角或銳角) 愈大,∠PAQ 愈小

3. 若 AO = PO ("A 在圓上"),則 ∠PAQ = 定值 (90°)

把以上結論應用於本題的 A, P, Q 所在平面 (屬於第 1 種情形):

cos∠PAQ 最小 (∠PAQ 最大) 時,AO 垂直 PQ。

cos∠PAQ 最大 (∠PAQ 最小) 時,即把前述 PQ 旋轉 90° 時 (考慮 AP 長與∠AOP 關係即知),且在此 A, P, Q 所在平面垂直 xy 平面 (若不易體會,可用三垂線定理證之)。

參考圖示h ttp://imgur.com/W0WPdrF


cos∠PAQ 的最大值: 如上圖左,tan∠PAQ = 4/5 (差角公式) ⇒ cos∠PAQ = 5/√41

cos∠PAQ 的最小值: 如上圖右,tan(∠PAQ/2) = 2/√14 ⇒ cos∠PAQ = 5/9,此時 aΔPAQ =  2√14


圖片附件: 101中正高中.png (2016-9-30 03:37, 5.25 KB) / 該附件被下載次數 5632
https://math.pro/db/attachment.php?aid=3631&k=84e470a905c6807c20e2d2b5145cec63&t=1714092907






歡迎光臨 Math Pro 數學補給站 (https://math.pro/db/) 論壇程式使用 Discuz! 6.1.0